Pulmonary Disorders Flashcards
Questions 1 and 2 pertain to the following case.
A 20-year-old woman presents to the clinic with an
asthma exacerbation. She states that she has been using
her boyfriend’s albuterol inhaler on a regular basis for
the past 2 years. During the past few months, she has
been using the inhaler on a daily basis and sometimes
at night.
1. Which description best classifies her asthma
severity?
A. Mild intermittent.
B. Mild persistent.
C. Moderate persistent.
D. Severe persistent.
- Answer: C
Answer A is incorrect because patients with mild inter-
mittent have symptoms 2 days or fewer per week. This
patient uses the inhaler throughout the day every day
and sometimes at night, indicating that she has daily
symptoms. Answer B is incorrect because the patient
is having daily symptoms, and patients with mild per-
sistent asthma have symptoms more often than 2 days
per week but fewer than daily. Answer C is correct
because the patient is having daily symptoms. Answer
D is incorrect because “severe persistent” means that
the frequency of symptoms is throughout the day,
nighttime symptoms often occur seven times a week,
a SABA is needed several times a day, normal activity
is extremely limited, the FEV1
is less than 60% of pre-
dicted, and the FEV1
/FVC is reduced by more than 5%.
- Which is the best asthma maintenance therapy for
this patient?
A. Fluticasone low dose.
B. Montelukast.
C. Fluticasone low dose plus salmeterol.
D. Fluticasone high dose.
- Answer: C
Moderate asthma requires step 3 treatment for control.
Step 3 preferred treatment is an inhaled steroid (low
dose) plus a LABA. Answer A is incorrect because low-
dose fluticasone is considered step 2 treatment. Answer
B is incorrect because montelukast is an alternative
treatment for step 2. Answer C is correct; fluticasone
low dose plus salmeterol is the preferred step 3 treat-
ment. Answer D is incorrect because high-dose flutica-
sone monotherapy is not recommended for treatment of
any step of asthma treatment.
- Which type of data best classifies the number of
times a low-dose inhaled corticosteroid/formoterol
inhaler is used in 1 month?
A. Nominal.
B. Ordinal.
C. Interval.
D. Ratio.
- Answer: D
Answer A is incorrect because nominal data contain no
quantitative value and are simply labels. Ordinal data is
a measure of non-numeric concepts in which the order
of the value is important but the differences between
values are not truly known, making Answer B incorrect.
Answer C is incorrect because although interval data
are ranked in a specific order with a consistent level of
magnitude difference between units, there is no absolute
zero. Answer D is correct because ratio data are ranked
in a specific order with a consistent level of magnitude
difference between units, with an absolute zero.
- You are designing a study in which you will com-
pare the percentages of patients with an asthma-
related hospitalization receiving fluticasone/
salmeterol and those receiving fluticasone alone.
Which statistical test is best for analyzing this
comparison?
A. Analysis of variance (ANOVA).
B. Chi-square.
C. Mann-Whitney U test.
D. Unpaired t-test.
- Answer: B
The percentage of patients receiving fluticasone/salme-
terol who have an asthma-related hospitalization will
be compared with the percentage of patients receiving
fluticasone who have an asthma-related hospitaliza-
tion. We assume that these two groups are normally
distributed. These data are considered nominal. Use
of ANOVA is appropriate when there are more than
two treatment groups; therefore, Answer A is incor-
rect. Answer B is correct because the chi-square test
is appropriate for analyzing nominal or categorical
data. The Mann-Whitney U test is appropriate when
continuous data are not normally distributed; therefore,
Answer C is incorrect. Answer D is incorrect because
the Student unpaired t-test is used for continuous data
that are normally distributed.
- A 22-year-old woman with asthma is taking an
albuterol metered-dose inhaler (MDI) 2 puffs as
needed and fluticasone 110 mcg/puff MDI 2 puffs
twice daily. She received the influenza vaccine
during last year’s influenza season, completed the
mRNA coronavirus disease 2019 (COVID-19) vac-
cine series, and her last tetanus vaccine (tetanus,
diphtheria, and pertussis [Tdap]) was at age 17;
there is no documentation of her having received a
pneumococcal vaccine. Which is the best vaccine
for her to receive at her next family medicine clinic
appointment, scheduled for July?
A. Influenza.
B. Pneumococcal.
C. Tetanus and diphtheria (Td).
D. Herpes zoster.
- Answer: B
Answer A is incorrect because the influenza vaccine
is recommended in people with chronic cardiovascular
or pulmonary diseases such as asthma. However, usu-
ally the influenza vaccine is given in the fall or early
winter to offer protection when the risk of infection is
highest. Answer B is correct; the pneumococcal vac-
cine is recommended in people 19–64 years of age with
asthma, and this patient falls into this category. Answer
C is incorrect because the tetanus booster (Td) is rec-
ommended every 10 years, and it has not been 10 years
since this patient’s last Td, which was given as Tdap.
The herpes zoster vaccine is recommended by the CDC
for adults 50 and older so Answer D is incorrect.
- A 60-year-old man is diagnosed with chronic
obstructive pulmonary disease (COPD) today. He
reports experiencing symptoms most days of the
week. His Modified Medical Research Council
(mMRC) score is 1. His spirometry reveals a forced
expiratory volume in 1 second (FEV1
) of 70% of
predicted and an FEV1
/forced vital capacity (FEV1
/
FVC) of 60% of predicted. He has had no previous
COPD exacerbations. Which medication is best to
initiate?
A. Inhaled fluticasone.
B. Inhaled tiotropium.
C. Inhaled fluticasone/salmeterol.
D. Oral roflumilast.
- Answer: B
This patient is in group A, based on having no exacerba-
tions and an mMRC score of 1. A single LA bronchodi-
lator is the best choice of medication treatment. Answer
A is incorrect because an ICS is considered only in
patient group E with elevated eosinophils and should
never be used as monotherapy in COPD. Tiotropium
is a LA bronchodilator (anticholinergic) that would be
appropriate in this patient, making Answer B correct.
(A LABA would also be appropriate, but it was not one
of the choices.) Answer C is incorrect because the ICS/
LABA combination is generally not preferred for treat-
ment of COPD. Answer D is incorrect because roflumi-
last is only indicated in severe COPD (FEV1 less than
50% of predicted) associated with chronic bronchitis
when the patient has a history of frequent exacerbations
- A 75-year-old woman with a 50 pack-year history of
tobacco use presents to her primary care physician
with a complaint of worsening shortness of breath
over the past 6 months. She has no documented
history of asthma or COPD. Which symptom is
most consistent with the diagnosis of COPD?
A. Dyspnea that is progressive, persistent, and
worse with exertion.
B. FEV1
/FVC greater than 70% of predicted.
C. Chronic cough present only at night.
D. COPD Assessment Test (CAT) score of 5.
- Answer: A
Answer A is correct because dyspnea that is progres-
sive, persistent, and worse with exertion is a hallmark
symptom of COPD. Answer B is incorrect because spi-
rometry findings of an FEV1
/FVC less than 70% of pre-
dicted are required for a diagnosis of COPD. Answer C
is incorrect because although a chronic cough is usually
a symptom of COPD, this cough is rarely only nocturnal
and is most often present throughout the day. The CAT
has a score range of 5–30, with higher scores indicat-
ing worse COPD. A CAT score of 5 is within the upper
limit of normal scores, making Answer D incorrect.
- A 68-year-old man was given a diagnosis of COPD
1 year ago, and his symptoms have been managed
with an albuterol inhaler as needed and umeclidin-
ium/vilanterol since. He reports adherence to cur-
rent regimen with appropriate inhaler technique.
Recent spirometry revealed an FEV1
/FVC less
than 70% and FEV1
of 65%. Last week, he was dis-
charged from the hospital after his second COPD
exacerbation in the past year. His current mMRC
score is 1 and blood eosinophil count is 212 cells/
mcL. Which maintenance therapy is most appro-
priate for this patient?
A. Continue umeclidinium/vilanterol.
B. Continue umeclidinium/vilanterol and add
roflumilast.
C. Continue umeclidinium/vilanterol and add
fluticasone.
D. Discontinue umeclidinium/vilanterol and start
fluticasone furoate/umeclidinium/vilanterol.
- Answer: D
This patient has had two exacerbations in the past year
on dual bronchodilator therapy and therefore should
follow the exacerbation pathway. The patient is cur-
rently taking a LABA/LAMA combination and should
be escalated to LABA/LAMA/ICS based on a blood
eosinophil count > 100 cells/mcL, making Answer D
correct. Answer A is incorrect because this option does
not escalate therapy. Answer B is incorrect because
roflumilast is considered if a blood eosinophil count is
< 100 cells/mcL, FEV1
is < 50%, and chronic bronchi-
tis is present. Answer C is incorrect because although
adding an ICS is appropriate, a single inhaler is pre-
ferred because it may be more convenient and effective.
In addition, a single ICS inhaler can potentially lead to
ICS monotherapy, which is not appropriate in patients
with COPD.
- A 32-year-old woman, 30 weeks pregnant, is seen
for a routine prenatal check today (October). This
is her second pregnancy in 3 years. During her
first pregnancy, she received Tdap at 36 weeks’
gestation; her last tetanus booster was at age 21.
She received all of her routine childhood immuni-
zations and is up to date with COVID-19 vaccines.
Which immunizations would be best for her today?
A. Td.
B. Td and influenza.
C. Tdap.
D. Tdap and influenza.
- Answer: D
During each pregnancy, it is recommended that the
woman receive a Tdap vaccination, regardless of the
time since her last Td or Tdap immunization. Therefore,
Answers A and B are incorrect because they do not
include Tdap. Answer C is incorrect because an annual
influenza vaccine is recommended for all pregnant
women, and the visit is taking place in October, when
it would be appropriate to administer the influenza vac-
cine. Answer D is correct because it includes both Tdap
and influenza.
- A 21-year-old man is seen for his annual wellness
exam in July. He received an influenza vaccine
last October but states that it was his first immu-
nization in about 10 years, besides completing the
mRNA COVID-19 vaccine series. He works full-
time at an office job that requires no travel. Which
vaccines would be best for him today?
A. Tdap, human papillomavirus (HPV).
B. Tdap, influenza, meningococcal polysaccharide.
C. Td, meningococcal polysaccharide, HPV.
D. Td, influenza.
- Answer: A
It is recommended that a one-time dose of Tdap be
substituted for Td in all patients and that the HPV vac-
cine be administered to males up to age 45. Therefore,
Answer A is correct. Answer B is incorrect because
it is inappropriate to administer the influenza vaccine
in July, and the patient has no risk factors indicating
that he would benefit from the meningococcal vaccine
(e.g., he is not living in a college dormitory and does not
travel frequently). Answer C is incorrect because the
patient should receive Tdap in place of the Td booster
and does not need the meningococcal vaccine. Answer
D is incorrect because the patient should receive Tdap
in place of the Td booster, and it is not appropriate to
administer the influenza vaccine in July
Patient Cases
Questions 1–3 pertain to the following case.
A 23-year-old woman has been coughing and wheezing about twice weekly, and she wakes up at night about three
times per month. She has never been given a diagnosis of asthma and has not been to a physician, she says, “in
years.” She uses her roommate’s albuterol inhaler, but having recently run out of refills, she is seeking care. Her
activities are not limited by her symptoms. Spirometry today reveals FEV1
82% of predicted.
1. According to the NAEPP guidelines, which is the best classification of her asthma?
A. Intermittent.
B. Mild persistent.
C. Moderate persistent.
D. Severe persistent.
- Answer: B
Her symptom frequency of twice weekly, her FEV1
of
more than 80% of predicted (normal), and the lack of
interference with activity are consistent with intermit-
tent asthma. Answer A is incorrect; her night awak-
enings for asthma symptoms occur three times per
month, which is consistent with mild persistent asthma.
Therefore, Answer B is correct. The specific level of
persistent asthma is based on the most severe category
met. Answers C and D are incorrect because only one
of her signs and symptoms falls under mild persistent
asthma (nighttime awakenings), and the rest fall under
intermittent asthma.
- According to GINA guidelines, which medication is best to recommend for her, in addition to budesonide/
formoterol MDI every 4–6 hours as needed?
A. No additional therapy needed.
B. Oral montelukast 10 mg/day.
C. Mometasone DPI 220 mcg/puff 2 puffs daily.
D. Budesonide/formoterol MDI 80/4.5 mcg per puff 2 puffs twice daily.
- Answer: A
Because she has mild persistent asthma, step 2 is rec-
ommended for initial treatment. Answer A is correct
given that step 2 treatment involves adding an as-needed
low-dose ICS-formoterol. Answer B is incorrect
because montelukast is an alternative therapy (not first
line) for step 2. Answer C is incorrect; mometasone 220
mcg 2 puffs once daily is a medium-dose ICS (alterna-
tive step 3). Budesonide/formoterol at the dose listed is
a low-dose ICS plus a LABA (step 3 therapy), making
Answer D incorrect.
- At first, the patient’s symptoms were well controlled on your recommended therapy. However, when winter
arrived, her symptoms were no longer well controlled, and she started using her budesonide/formoterol MDI
3 or 4 days per week during the day. Which is the preferred treatment change?
A. No change in therapy is needed.
B. Add budesonide/formoterol MDI 80/4.5 mcg per puff 2 puffs twice daily.
C. Add montelukast orally 10 mg daily.
D. Add mometasone DPI to 220 mcg/puff 2 puffs daily.
- Answer: B
Her asthma is classified as not well controlled because
of the frequency of her daytime symptoms and her need
to use albuterol more than 2 days/week. When asthma
is classified as not well controlled, the recommendation
is to increase to the next step of treatment. The patient
would now be classified as needing step 3 treatment.
The recommended treatment for step 3 is a low-dose
ICS-formoterol. Answer A is incorrect because the rec-
ommended treatment for step 3 is a daily low-dose ICS
plus a LABA. Answer B is correct because it results in
single maintenance and reliever therapy (SMART) with
ICS-formoterol reliever (step 3 therapy). Adding mon-
telukast is an alternative therapy for step 2 and would
not be the preferred option, making Answer C incor-
rect. Answer D is incorrect; mometasone 220 mcg/puff
2 puffs daily is a medium-dose ICS.
- A 25-year-old man presents to the ED with shortness of breath at rest. He is having trouble with conversation.
He used 4 puffs of albuterol MDI at home, with no resolution of symptoms. His FEV1
is 38% of predicted.
Which therapy, in addition to oxygen, is best for him initially in the ED?
A. Oxygen alone is sufficient.
B. Give albuterol MDI 8 puffs every 20 minutes for 1 hour.
C. Give albuterol plus ipratropium by nebulizer every 20 minutes for 1 hour, plus oral corticosteroids.
D. Give albuterol plus ipratropium by nebulizer every 20 minutes for 1 hour, plus intravenous corticosteroids.
- Answer: C
Because the patient is experiencing shortness of breath at
rest, has trouble with conversation, and has an FEV1
less
than 50% of expected, his asthma exacerbation is clas-
sified as severe. For severe asthma exacerbations in the
ED setting, pharmacologic treatment is recommended
in addition to oxygen to achieve an Sao2
of 93%–98%;
therefore, Answer A is incorrect. Answer B is incorrect
because a high-dose inhaled SABA should be combined
with ipratropium and administered by either a nebulizer
or an MDI with valved holding chamber every 20 min-
utes for 1 hour or continuously for severe asthma exac-
erbations in the ED setting. Answer C is correct because
it correctly combines a SABA with ipratropium and
includes OCSs. Answer D correctly provides a SABA
with ipratropium; however, intravenous corticosteroids
are not recommended unless the patient has an incom-
plete response to inhaled and oral treatments
Patient Cases
5. A 62-year-old man was recently given a diagnosis of COPD. Spirometry reveals an FEV1
/FVC 60% of pre-
dicted, pre-bronchodilator FEV1
70% of predicted, and post-bronchodilator FEV1
72% of predicted. His symp-
toms are quite bothersome. He has an mMRC score of 2. He had 1 exacerbation in the past year that did not
require hospitalization. In addition to albuterol 2 puffs every 4–6 hours as needed, which pharmacotherapy
option is most appropriate to initiate?
A. Tiotropium 2.5 mg 2 puffs once daily.
B. Salmeterol 50 mcg 1 puff twice daily.
C. Tiotropium/olodaterol 2.5/2.5 2 puffs once daily.
D. Fluticasone furoate/umeclidinium/vilanterol 100 mcg/62.5 mcg/25 mcg 1 inhalation once daily.
- Answer: C
Answer A is incorrect; according to the GOLD guide-
lines, the recommended treatment for patient group B
is the combination of a LABA and LAMA in addition
to a SA bronchodilator as needed. Answer B incorrect
because it does not include a LAMA in combination
with the salmeterol. Answer C correct because the
LABA/LAMA combination is recommended for initial
treatment of group B. Answer D is incorrect because
ICSs are only considered as part of triple therapy
(LABA + LAMA + ICS) in group E who have a blood
eosinophil count ≥ 300 cells/mcL.